Laura Damone
Thanks Received: 94
Atticus Finch
Atticus Finch
 
Posts: 468
Joined: February 17th, 2011
 
 
 

Q16 - Journalist: When judges do not maintain

by Laura Damone Fri Nov 06, 2020 5:48 pm

Question Type:
Weaken

Stimulus Breakdown:
Conclusion: Whenever lawyers try to influence jury verdicts by using inflammatory language and badgering witnesses, it is reasonable to doubt whether the verdict is correct.

Premise: Using inflammatory language and badgering witnesses hinder the jury's effort to reach a correct verdict.

Background: When judges do not maintain strict control over their courtrooms, lawyers often use inflammatory language and badger witnesses in an effort to influence the jury's verdict.

Answer Anticipation:
Whenever a conclusion on an Assumption Family question introduces a new concept, that represents a gap in reasoning. Here, the conclusion talks about it being reasonable to doubt the accuracy of a verdict, but the premise doesn't! Reading like a debater, I accept that lawyers use these obstructive tactics to hinder the jury's efforts, but I don't believe that proves that we can doubt the correctness of their verdicts, because that assumes that the lawyers' efforts at hinderance are successful. Predict an answer that attacks that gap in reasoning.

Correct answer:
D

Answer choice analysis:
(A) A classic trap! When an argument tells us that NOT X is correlated with Y, we can't weaken that argument by saying that X is correlated with Y. Those two things aren't mutually exclusive! Eliminate.

(B) Nope. This irrelevant comparison between lawyers and judges doesn't impact our argument, and there's a slight detail creep from "verdict is correct" to "verdict is just."

(C) This one starts out strong: you could definitely craft a correct answer about people who are influenced by inflammatory language. But, it goes awry when it claims that they're unlikely to admit being influenced. Who cares? We don't need them to admit it for our argument to work.

(D) D is correct but it's SO hard to pick. Thank goodness for wrong-to-right because that's the key to this question. D looks really good at the outset, matching our prephrase and attacking the assumption that the obstruction is actually effective. But it lost me when it got narrow, applying only to cases in which the jury is also presented with legitimate evidence. At this point, I deferred, went on to E, eliminated E, and came back to D. Even though it's narrow, and that's not a desirable feature in a Weaken answer, it's still the best of the five answers because it attacks the assumption.

(E) Jury selection? Bias? No way. Out of scope.

Takeaway/Pattern:
Sometimes, test writers make a question hard by crafting a super tempting wrong answer that competes with the right answer. Other times, they make a question hard by making all five answers look wrong, forcing you to pick the best of the lot, even though it's not perfect. Be prepared for that, especially in Qs 15-23. There's a hierarchy of flaws - some are grounds for dismissal while others are red flags but not necessarily fatal errors. The real challenge comes when something that could be grounds for dismissal in one question is given a pass in another. As a general rule, be wary of Weaken answers that are significantly more narrow than the argument you're weakening. But when compared to the irrelevance of the other answer choices, in this case, we have to let it slide.

#officialexplanation
Laura Damone
LSAT Content & Curriculum Lead | Manhattan Prep